Block 9 Pharm Antiadrenegic W Exp Prt 1

Approved & Edited by ProProfs Editorial Team
The editorial team at ProProfs Quizzes consists of a select group of subject experts, trivia writers, and quiz masters who have authored over 10,000 quizzes taken by more than 100 million users. This team includes our in-house seasoned quiz moderators and subject matter experts. Our editorial experts, spread across the world, are rigorously trained using our comprehensive guidelines to ensure that you receive the highest quality quizzes.
Learn about Our Editorial Process
| By Chachelly
C
Chachelly
Community Contributor
Quizzes Created: 513 | Total Attempts: 592,897
Questions: 15 | Attempts: 357

SettingsSettingsSettings
Block 9 Pharm Antiadrenegic W Exp Prt 1 - Quiz

Questions and Answers
  • 1. 

    This drug is a partial agonist at beta-1 and beta-2 receptors

    • A.

      Atenolol

    • B.

      Esmolol

    • C.

      Metyrosine

    • D.

      Phenoxybenzamine

    • E.

      Pindolol

    • F.

      Propranolol

    • G.

      Sotalol

    • H.

      Tamsulosin

    • I.

      Timolol

    Correct Answer
    E. Pindolol
    Explanation
    Pindolol is a partial agonist at beta-1 and beta-2 receptors. This means that it binds to these receptors and activates them, but with less efficacy compared to a full agonist. As a result, it produces a weaker response compared to a full agonist. This property of pindolol can be beneficial in certain situations where a moderate level of receptor activation is desired, such as in the treatment of hypertension or angina.

    Rate this question:

  • 2. 

    This drug is a selective beta-1 antagonist frequently for the chronic treatment of atrial fibrillation

    • A.

      Atenolol

    • B.

      Esmolol

    • C.

      Metyrosine

    • D.

      Phenoxybenzamine

    • E.

      Pindolol

    • F.

      Propranolol

    • G.

      Sotalol

    • H.

      Tamsulosin

    • I.

      Timolol

    Correct Answer
    A. Atenolol
    Explanation
    Atenolol is a selective beta-1 antagonist, meaning it specifically blocks the beta-1 receptors in the body. It is commonly used for the chronic treatment of atrial fibrillation, a condition characterized by irregular and rapid heartbeats. By blocking the beta-1 receptors, atenolol helps to slow down the heart rate and reduce the force of contractions, thus improving the symptoms associated with atrial fibrillation. Other drugs listed as options do not have the same selectivity for beta-1 receptors or are not commonly used for the treatment of atrial fibrillation.

    Rate this question:

  • 3. 

    This drug is a beta-blockers that can also blocks potassium channels

    • A.

      Atenolol

    • B.

      Esmolol

    • C.

      Metyrosine

    • D.

      Phenoxybenzamine

    • E.

      Pindolol

    • F.

      Propranolol

    • G.

      Sotalol

    • H.

      Tamsulosin

    • I.

      Timolol

    Correct Answer
    G. Sotalol
    Explanation
    Sotalol is a beta-blocker that is known to block potassium channels in addition to its beta-blocking effects. This dual action makes it useful in the treatment of certain cardiac arrhythmias, as blocking potassium channels helps to prolong the action potential duration and prevent re-entry of electrical signals. The other options listed are also beta-blockers, but they do not have the additional effect of blocking potassium channels.

    Rate this question:

  • 4. 

    This drugs is frequently used in case of prostatic hyperplasia

    • A.

      Atenolol

    • B.

      Esmolol

    • C.

      Metyrosine

    • D.

      Phenoxybenzamine

    • E.

      Pindolol

    • F.

      Propranolol

    • G.

      Sotalol

    • H.

      Tamsulosin

    • I.

      Timolol

    Correct Answer
    H. Tamsulosin
    Explanation
    Tamsulosin is frequently used in cases of prostatic hyperplasia. Prostatic hyperplasia refers to the enlargement of the prostate gland, which can cause urinary problems. Tamsulosin is an alpha-blocker medication that helps relax the muscles in the prostate and bladder neck, making it easier to urinate. It specifically targets the alpha-1 receptors in the prostate, leading to improved urine flow and reduced symptoms associated with prostatic hyperplasia. Therefore, Tamsulosin is the correct answer for this question.

    Rate this question:

  • 5. 

    This drug is sometimes used in cardiovascular emergencies by IV infusion because of its extremely short half life

    • A.

      Atenolol

    • B.

      Esmolol

    • C.

      Metyrosine

    • D.

      Phenoxybenzamine

    • E.

      Pindolol

    • F.

      Propranolol

    • G.

      Sotalol

    • H.

      Tamsulosin

    • I.

      Timolol

    Correct Answer
    B. Esmolol
    Explanation
    Esmolol is sometimes used in cardiovascular emergencies by IV infusion because of its extremely short half-life. This means that it is rapidly eliminated from the body, allowing for quick adjustments to the dosage if needed. In emergency situations, it is important to have a drug that can be easily controlled and adjusted, and esmolol provides this advantage.

    Rate this question:

  • 6. 

    Which of the following is a pharmacological effects of alpha-1 receptor antagonists?

    • A.

      Reversal of norepinephrine-induced hypertension

    • B.

      Reversal of epinephrine-induced hypertension

    • C.

      Decreased skin blood flow

    • D.

      Decreased heart rate

    • E.

      Contraction of trigone and bladder internal sphincter

    • F.

      Contraction of the radial muscle of iris.

    Correct Answer
    B. Reversal of epinephrine-induced hypertension
    Explanation
    Answer: B
    Epinephrine activates alpha-1 receptors so causing vasoconstriction, and beta-2 receptors, so
    causing vasodilation. Alpha-1 receptor mediated vasoconstriction and the epinephrine-induced
    increased stroke volume cause an increase in mean blood pressure. By blocking alpha-1
    receptors the beta-2 mediated vasodilating activity of epinephrine is unopposed and blood
    pressure falls.
    A) The hypertensive effect of norepinephrine is decreased, but not reversed because
    norepinephrine has no vasodilating activity (it does not activate beta-2 receptors).
    C) By blocking alpha-1 receptors, skin vessels are relaxed and blood flow through the skin is
    usually increased, not decreased.
    D) Blockade of alpha-1 receptors causes no direct effects upon the heart. However the
    vasodilation may cause reflex tachycardia. Therefore heart rate is either minimally affected or
    increased.
    E) Alpha-1 receptors are located on smooth muscle of the trigone and bladder internal
    sphincter. By blocking these receptors relaxation, not contraction, of these muscles occurs.
    F) Activation of alpha-1 receptors contracts the radial muscle of iris, so causing mydriasis. By
    blocking alpha-1 receptors, alpha antagonist cause relaxation, not contraction.

    Rate this question:

  • 7. 

    Which of the following is the most likely adverse effect of chronic treatment with selective alpha-1 blockers?

    • A.

      Hypertension

    • B.

      Bradycardia

    • C.

      Digital vasospasm

    • D.

      Sodium and water retention

    • E.

      Urinary retention

    Correct Answer
    D. Sodium and water retention
    Explanation
    Answer: D
    A chronic treatment with selective alpha-1 blockers may cause hypotension. This hypotension
    usually elicits compensatory effects through actions not dependent on adrenergic activation
    (namely Na+ retention and expansion of blood volume).
    A) Hypotension, not hypertension, is a likely adverse effects of alpha-1 blockers.
    B) By causing hypotension these drugs tend to cause reflex tachycardia, not bradycardia.
    C) Alpha-1 blockers cause peripheral vasodilation, so digital vasospasm is quite unlikely.
    E) Alpha-1 blockers relax the internal sphincter of the bladder ,so urinary retention is unlikely.

    Rate this question:

  • 8. 

    In an experimental animal model, acute prazosin pretreatment will most effectively counteract which of the following drug-induced effects?

    • A.

      Isoproterenol-induced tachycardia

    • B.

      Nicotine-induced sweating

    • C.

      Dopamine-induced increase in renal blood flow

    • D.

      Phenylephrine-induced hypertension

    • E.

      Pilocarpine-induced contraction of ciliary muscle

    Correct Answer
    D. Phenylephrine-induced hypertension
    Explanation
    Answer: D
    Prazosin is a selective alpha-1 blocker and therefore will most effectively counteract the effect
    of a selective alpha-1 agonist like phenylephrine
    A) Isoproterenol induces tachycardia by activating beta-1 and (to a lesser extent) beta-2
    receptors. The blockade of alpha-1 receptors cannot antagonize this effect.
    B) Nicotine induces sweating by activating Nn receptors in ganglia which in turn increases
    sympathetic firing. However the sympathetic fibers innervating sweat glands are cholinergic
    and therefore, the blockade of alpha-1 receptors cannot antagonize this effect.
    C) Dopamine increases renal blood flow by activating D1 receptors. By blocking alpha-1
    receptors prazosin can also increases renal blood flow.
    E) Pilocarpine contracts the ciliary muscle by activating M3 receptors. The blockade of alpha-1
    receptors cannot antagonize this effect.

    Rate this question:

  • 9. 

    Prazosin is contraindicated, or should be used with caution, in which of the following diseases?

    • A.

      Systemic mastocytosis

    • B.

      Prostate cancer

    • C.

      Chronic hypertension

    • D.

      Chronic heart failure

    • E.

      Raynaud’s disease

    Correct Answer
    A. Systemic mastocytosis
    Explanation
    Answer: A
    Since alpha-1 blockers cause vasodilation and impair the baroreceptor reflex, they are
    contraindicated in all disorders leading to orthostatic hypotension, including systemic
    mastocytosis. In fact in this disease the large number of mast cells causes an elevated plasma
    histamine concentration, which in turn leads to hypotension and even to cardiovascular
    collapse.
    B, C, D, E) All these disease are actually indications, not contraindications, for the therapeutic
    use of prazosin.

    Rate this question:

  • 10. 

    An IV injection of norepinephrine was given to a laboratory animal before and after the administration of a new drug X, and the mean blood pressure recorded. The results are depicted in the following figure. Which of the following drugs does the new agent most closely resemble?

    • A.

      Albuterol

    • B.

      Cocaine

    • C.

      Labetalol

    • D.

      Phenylephrine

    • E.

      Propranolol

    • F.

      Dopamine

    Correct Answer
    C. Labetalol
    Explanation
    Answer: C
    The blood pressure depends on cardiac output and total peripheral resistance according to the
    formula: BP = SV x HR x TPR. In order to completely abolish the action of norepinephrine the
    drug must antagonize the increased stoke volume (mediated by the activation of beta
    receptors) and the increased total peripheral resistance (mediated by the activation of alpha-1
    receptors). Therefore the drug must have both alpha and beta blocking activity, like labetalol.
    A, B, D, E, F) (see explanation above)

    Rate this question:

  • 11. 

    Which of the following is a metabolic effect of non-selective beta blockers?

    • A.

      Enhancement of insulin-induced hypoglycemia

    • B.

      Increased lipolysis

    • C.

      Increased renin release

    • D.

      Increased hepatic glycogenolysis

    • E.

      Increased HDL

    Correct Answer
    A. Enhancement of insulin-induced hypoglycemia
    Explanation
    Answer: A
    Gluconeogenesis and glycogenolysis are beta-2 mediated effects. By blocking these effects
    non-selective beta-blockers can enhance hypoglycemia induced by insulin (or other
    hypoglycemic drugs) in diabetic patients.
    C) Renin release can be stimulated by the activation of beta-1 receptors in the juxtaglomerular
    cells of macula densa. All beta-blockers decrease, not increase, renin release.
    B) Beta blockers actually decrease lipolysis and usually reduce the release of free fatty acid
    from adipose tissue.
    D, E) Beta blockers actually decrease hepatic gluconeogenesis and plasma levels of HDL.

    Rate this question:

  • 12. 

    In an experimental animal model, an acute propranolol pretreatment will most effectively counteract which of the following drug-induced effects?

    • A.

      Isoproterenol-induced hyperglycemia

    • B.

      Nicotine-induced decrease in skin blood flow

    • C.

      Norepinephrine-induced reflex bradycardia

    • D.

      Phenylephrine-induced mydriasis

    • E.

      Pilocarpine-induced contraction of bronchial muscle

    Correct Answer
    A. Isoproterenol-induced hyperglycemia
    Explanation
    Answer: A
    Isoproterenol-induced hyperglycemia is mainly due to beta-2 receptor mediated
    gluconeogenesis and glycogenolysis, and therefore it can be effectively counteracted by
    propranolol, which block beta-2 receptors. All the other listed actions cannot be antagonized
    by propranolol because they are not mediated by activation of beta receptors.
    B) Nicotine-induced activation of Nn receptors in ganglia increases the firing of postganglionic
    sympathetic nerves, which in turn cause skin vasoconstriction by activating alpha-1 receptors
    C) Norepinephrine-induced reflex bradycardia is mediated by the activation of M2 cardiac
    receptors.
    D) Phenylephrine-induced mydriasis is mediated by the activation of alpha-1 receptors
    E) Pilocarpine-induced contraction of bronchial musculature is mediatd by the activation of M3
    receptors.

    Rate this question:

  • 13. 

    Which of the following pairs of autonomic drugs is a correct receptor agonistantagonist match?

    • A.

      Isoproterenol - prazosin

    • B.

      Epinephrine - labetalol

    • C.

      Phenylephrine - atropine

    • D.

      Albuterol - Atenolol

    • E.

      Nicotine - dopamine

    • F.

      Norepinephrine - clonidine

    Correct Answer
    B. Epinephrine - labetalol
    Explanation
    Answer: B
    Epinephrine activates alpha-1, alpha-2, beta-1 and beta-2 receptors,. labetalol is able to block
    all these receptors. Therefore it will be able to antagonize all the effects of epinephrine.
    A, C, D, E ,F) All these drug pairs act on different receptors and therefore are not a receptor
    agonist-antagonist match.

    Rate this question:

  • 14. 

    A 75-year-old man, suffering from congestive heart failure, has been recently diagnosed with benign prostatic hyperplasia. Since surgical ablation of the prostate was contraindicated, a drug suitable for the treatment of patient’s impaired bladder emptying was prescribed. Which of the following statements best explains the molecular mechanism of action of that drug?

    • A.

      Blockade of beta-2 receptors

    • B.

      Activation of beta-2 receptors

    • C.

      Activation of alpha-2 receptors

    • D.

      Blockade of alpha-1 receptors

    • E.

      Activation of M3 receptors

    Correct Answer
    D. Blockade of alpha-1 receptors
    Explanation
    Answer: D
    Selective alpha-1 blockers (like prazosin, tamsulosin etc.) are the drugs of choice for the
    treatment of impaired bladder emptying in prostatic hyperplasia. By blocking alpha-1 receptors
    these drugs relax the prostate capsule and the internal sphincter of the bladder (which have
    plentiful alpha-1 receptors), so allowing a better urine flow.
    A, B) Beta receptors are not significantly involved in the regulation of the internal sphincter of
    the bladder.
    C) Even if activation of alpha-2 receptors would cause a decrease of norepinephrine release,
    the action is too weak to be clinically significant.
    E) Activation of M3 receptors would cause a contraction of the detrusor muscle of the bladder,
    which would increase the urgency to void without improving voiding because of the urethral
    compression due to prostate hyperplasia.

    Rate this question:

  • 15. 

    A 27-year-old woman presented to the outpatient clinic with a 5-day history of left hand and arm pain. She noted that the fingers of her left hand became cold and somewhat blue especially when exposed to cold. Physical examination showed that patient’s extremities did have appropriate sensation. The hands were placed in cold water and it was noticed that several white splotches appeared and tingling was felt in the hands. Which of the following drugs would be appropriate to treat the patient’s disorder?

    • A.

      Neostigmine

    • B.

      Propranolol

    • C.

      Prazosin

    • D.

      Clonidine

    • E.

      Labetalol

    Correct Answer
    C. Prazosin
    Explanation
    Answer: C
    The signs and symptoms of the patient suggest the diagnosis of Raynaud’ disease.
    In this disease, which is idiopathic and is most common in young women, the threshold for
    vasospastic response is lowered by local cold or anything that activates the sympathetic
    outflow or release of catecholamines. By blocking alpha-1 receptors, prazosin is often helpful
    in reducing vasospastic episodes.
    A, B, D, E) There is no rationale for the use of all the other listed drugs in Raynaud’ disease.

    Rate this question:

Quiz Review Timeline +

Our quizzes are rigorously reviewed, monitored and continuously updated by our expert board to maintain accuracy, relevance, and timeliness.

  • Current Version
  • Mar 20, 2023
    Quiz Edited by
    ProProfs Editorial Team
  • Jun 03, 2012
    Quiz Created by
    Chachelly
Back to Top Back to top
Advertisement
×

Wait!
Here's an interesting quiz for you.

We have other quizzes matching your interest.